- Sat Jan 21, 2012 12:00 am
#41668
Complete Question Explanation
(The complete setup for this game can be found here: lsat/viewtopic.php?t=4937)
The correct answer choice is (B)
Answer choice (A) is incorrect because it violates the third rule. Answer choice (C) is incorrect because it K cannot work on Wednesday (O is already there). Answer choice (D) is incorrect because it violates the last rule. Answer choice (E) is incorrect because no supervisor would be assigned to Tuesday. Thus, answer choice (B) is correct (and, in answer choice (B), P would also have to work on Monday).
(The complete setup for this game can be found here: lsat/viewtopic.php?t=4937)
The correct answer choice is (B)
Answer choice (A) is incorrect because it violates the third rule. Answer choice (C) is incorrect because it K cannot work on Wednesday (O is already there). Answer choice (D) is incorrect because it violates the last rule. Answer choice (E) is incorrect because no supervisor would be assigned to Tuesday. Thus, answer choice (B) is correct (and, in answer choice (B), P would also have to work on Monday).